What degree of rotation about the origin will cause the triangle below to map
onto itself?
A.90
B.270
C.360
D.180

Answers

Answer 1
Sorry I don’t see any images or diagrams I’m not sure
Answer 2
360° Would cause the origin to map back onto itself, C. Hope this helps!

Related Questions

What’s the equation of the line that passes through the point (-4,4) and has a slope of 3/4

Answers

Answer:

y-y1=m(x-x1)

y-4=3/4(x+4)

y=3/4x+7

Decrease £19064.67 by 9.5%

Give your answer rounded to 2 DP. (decimal place)

Please Help !!!

Answers

Answer:

17253.52

Step-by-step explanation:

100-9.5=90.5

19064.67*0.905=17253.52

Find the height h of the triangle.

Answers

Answer:

A, 12 times the square root of 3

Step-by-step explanation:

24 sqaured plus 12 squared=432

square root of 432=20.9

12 times the sqaure root of 3=20.9

A cinema is doing a promotion to celebrate their 50th anniversary for 1 week. They give

away a free drink to every 98th customer, a free bag of popcorn to every 112th customer and

a free cinema ticket to every 224th customer. Which lucky customer will be the first to

receive all 3 items?​

Answers

Answer:

1,568 customer

Step-by-step explanation:

Find the lowest common multiple of 98, 112, and 224

98 = 98, 196, 294, 392, 490, 588, 686, 784, 882, 980, 1078, 1176, 1274, 1372, 1470, 1568, 1666

112 = 112, 224, 336, 448, 560, 672, 784, 896, 1008, 1120, 1232, 1344, 1456, 1568, 1680, 1792, 1904

224 = 224, 448, 672, 896, 1120, 1344, 1568, 1792, 2016, 2240

The lowest common multiple of 98, 112, and 224 is 1568

Therefore, the 1,568th customer will be the first to receive all 3 iitem

Hello, Brainly community!

This question is for all of those Calculus people out there.

The volume of a swimming pool is changing with respect to time, such that the volume is given by W(t), where W(t) is measured in cubic centimeters and t is measured in seconds. A tangent line is shown for W(t) at t = 3 seconds. Determine the best estimate for the value of the instantaneous rate of change of W(t) when t = 3.
(I've narrowed down the answer choices to 2, and just really need to find the right way of thinking to find the answer)

(A) W(lim t) as t goes to 3.
(B) [W(3.1) - W(2.9)] / 0.2.

Thank you in advance!

Answers

Answer:

(B)  [tex]\displaystyle \frac{W(3.1) - W(2.9)}{0.2}[/tex]

General Formulas and Concepts:

Calculus

Limits

Derivatives

The definition of a derivative is the slope of the tangent line.

Derivative Notation

Instantaneous Rates

Tangent Line: [tex]\displaystyle f'(x) = \frac{f(b) - f(a)}{b - a}[/tex]

Step-by-step explanation:

Since we are trying to find a rate at which W(t) changes, we must find the derivative at t = 3.

We are given 2 close answer choices that would have the same numerical answer but different meanings:

(A)  [tex]\displaystyle \lim_{t \to 3} W(t)[/tex](B)  [tex]\displaystyle \frac{W(3.1) - W(2.9)}{0.2}[/tex]

If we look at answer choice (A), we see that our units would simply just be volume. It would not have the units of a rate of change. Yes, it may be the closest numerically correct answer, but it does not tell us the rate at which the volume would be changing and it is not a derivative.

If we look at answer choice (B), we see that our units would be cm³/s, and that is most certainly a rate of change. Answer choice (B) is also a derivative at t = 3, and a derivative tells us what rate something is changing.

∴ Answer choice (B) will give us the best estimate for the value of the instantaneous rate of change of W(t) when t = 3.

Topic: AP Calculus AB/BC (Calculus I/I + II)

Unit: Differentiation

Book: College Calculus 10e

A statistics professor asked students in a class their ages. On the basis of this information, the professor states that the average age of all the students in the university is 24 years. This is an example of

Answers

Answer:

propbability ???

Step-bp explanation:

Answer:

Step-by-step explanation:

This is an example of a statistical mean.

The formula for the circumference of a circle is R = c/2(pi)

Find the radius of a circle that has a circumference of 16(pi)

A) r = 4
B) r = 8
C) r = 12
D) r = 16

Answers

The answer is B) r= 8

jordan wants to save to buy a car and decides to open a banking account that is offering an interest rate of 4.5% compounded annually how much will jordan have in the account after 5 years it he deposits $7,000 today?

Answers

Answer:

8,723.27$

Step-by-step explanation:

PLS HELP SOON WILL MARK BRAINLYEST

A railroad tunnel is shaped like a semi-ellipse, as shown below. A semiellipse is shown on the coordinate plane with vertices on the x axis and one point of intersection with the positive y axis. The height of the tunnel at the center is 35 ft, and the vertical clearance must be 21 ft at a point 8 ft from the center. Find an equation for the ellipse.

Answers

According to the question

b= 35 and (8,21) lies on the ellipse

After calculation we get a= 10

equation for the ellipse.

[tex] \frac{ {x}^{2} }{100} + \frac{ {y}^{2} }{1225} = 1[/tex]

The cost for 2 adults and 5 children to eat at the local buffet restaurant is \$46.50 . The cost for 1 adult and 1 child is $15. What is the cost of a child's meal ?

Answers

Answer:

c = 5.5

Step-by-step explanation:

2 a + 5 c = 46.50

a + c = 15

a = 15 - c

2(15-c ) +5c = 46.50

30 - 2c +5c = 46.5

3c = 16.5

c = 5.5

Let a be the adults and c be the children.

For the first equation there are 2 adults and 5 children so the equation is:-

2a + 5c = 46.50

And for the second equation there is 1 adult and 1 child. So the equation is:-

a + c = 15

a = 15 - c

You take this formula and substitute it to the above formula (2a + 5c = 46.50)

➡️ 2a + 5c = 46.50

➡️ 2(15 - c) + 5c = 46.50

➡️ 30 -2c + 5c = 46.50

➡️ 5c - 2c = 46.50 - 30
(You collect like terms)

➡️ 3c = 16.5

➡️ c = 16.5/3

➡️ c = 5.5

find the slope of the line passing through the points (-2,5) and (3/2,2)

Answers

Answer:

slope = - [tex]\frac{6}{7}[/tex]

Step-by-step explanation:

Calculate the slope m using the slope formula

m = [tex]\frac{y_{2}-y_{1} }{x_{2}-x_{1} }[/tex]

with (x₁, y₁ ) = (- 2, 5) and (x₂, y₂ ) = ([tex]\frac{3}{2}[/tex], 2)

m = [tex]\frac{2-5}{\frac{3}{2}-(-2) }[/tex]

   = [tex]\frac{-3}{\frac{3}{2}+2 }[/tex]

   = [tex]\frac{-3}{\frac{7}{2} }[/tex]

  = - 3  × [tex]\frac{2}{7}[/tex]

  = - [tex]\frac{6}{7}[/tex]

Select the expression that represents the following statement: add 24 to the quotient of 16 and 8.

Answers

Answer:

16/8 + 24

Step-by-step explanation:

find the domain of f(x)=sec(2x)

Answers

Answer:

*Refer the image attached

Step-by-step explanation:

*Refer the image attached


Which expressions are equivalent to 6 - 4.5+ (-8) ?
A 6 + 4.5 + 8
B 6 + 4.5 - 8
C 6 – 4.5'+8
D 6 - 4.5 - 8
E 6+(-4.5) - 8
F 6+(-4.5) + (-8).
G 6+ (-4.5) - (-8)

Answers

Answer:

I'm pretty sure the answers are options D , E and F .

Step-by-step explanation:

So here's how I did it , I got the original expression , solved it and the answer I found is -6.5. So then I solved all the others and D , E and F gave me -6.5.

Hope this helps lol

Which expressions are equivalent c and e are equivalent to 6-4.5+(-8)

Nathan is 1.55 meters tall. At 1 p.m., he measures the length of a tree's shadow to be 38.15 meters. He stands 32.9 meters away from the tree, so that the tip of his shadow meets the tip of the tree's shadow. Find the height of the tree to the nearest hundredth of a meter .

Answers

Answer:

11.26 m

Step-by-step explanation:

The height of the tree is about 11.25 meters.

What are similar triangles?

When the respective sides are proportional and the corresponding angles are congruent, two triangles are said to be similar.

Given that, the height of the person is 1.55 meters, the length of the tree's shadow is 38.15 meters, and the distance between the person and the tree is 32.9 meters.

Let the height of the tree be x.

Note that the scenario makes two similar triangles.

Since the ratio of the side lengths of similar triangles is proportional, it follows:

(38.15 - 32.9)/1.55 = 38.15/x

5.25/1.55 = 38.15/x

3.39 = 38.15/x

x = 38.15/3.39

x = 11.25

Hence, the height of the tree is about 11.25 meters.

Learn more about similar triangles: https://brainly.com/question/25882965

#SPJ2

Which operation will solve the following word problem? Jeff earns $14.00 per hour, Tom earns half as much as Jeff. How much does Tom earn per hour?


Multiplication


Subtraction


Addition


Division

Answers

Answer:

The correct option is (d).

Step-by-step explanation:

Given that,

Jeff earns $14.00 per hour.

Tom earns half as much as Jeff.

We need to find the amount earn by Tom per hour.

Tom's amount = Jeff's amount/2

So,

[tex]T=\dfrac{14}{2}\\\\T=\$7[/tex]

So, Tom earn $7 per hour. Hence, division operation is used. Jeff's amount is divided by 2.

5) On each birthday Rosa gets as many roses as she is old in years. She still has all the dried flowers and there are now 120 of them. How old is she? A) 10 B) 12 C) 14 D) 15 E) 20

Answers

Answer:

D) 15

Step-by-step explanation:

This is an arithmatic progression.

The formula for the sum of arithmatic progression is

[tex]s = \frac{n}{2} (2a + (n - 1)d)[/tex]

where d is the common difference between successive terms and a is the first term. By applying this formula,

[tex]120 = \frac{n}{2} (2(1) + (n - 1)(1)) \\ 120 = \frac{n}{2} (1 + n) \\ n(1 + n) = 240 \\ n {}^{2} + n - 240 = 0 \\ (n - 15)(n + 16) = 0 \\ n = 15 \: or \: n = - 16(reject)[/tex]

Greg buys 60 garden plants at a cost price of $2.00 each to sell in his shop. He sells 25 of them at the profit of 75% and 18 of them at the profit of 35%. He sells the rest of the plants for 4/5 of the cost price calculate the profit or loss he makes from selling 60 plants stating if it is a profit or loss

Answers

Answer:

$43.30 profit

Step-by-step explanation:

Total cost of plant:

60*2 = 120

Greg makes total of:

25*(2 + 0.75*2) + 18*(2 + 0.35*2) + (60 - 25 - 18)*2*4/5 = 163.3

Since Greg mare than cost, he has a profit and the amount is:

163.3 - 120 = 43.3

2 1/4 x 3 1/5 brainliest

Answers

Answer:

36/5

Step-by-step explanation:

9/4×16/5

144/20

36/5

hope this is helpful

Answer:

[tex]7\frac{1}{5}[/tex]

Step-by-step explanation:

1. start by turning the fractions improper fractions:

[tex]2\frac{1}{4} =\frac{9}{4}[/tex]

[tex]3\frac{1}{5} =\frac{16}{5}[/tex]

2. then multiply them together:

[tex]\frac{9}{4}[/tex] x [tex]\frac{16}{5}[/tex] = [tex]\frac{144}{20}[/tex]

3. then simplify the fraction:

[tex]\frac{144}{20}[/tex][tex]=\frac{36}{5}[/tex]

4. turn it into a proper fraction:

[tex]\frac{36}{5} =7\frac{1}{5}[/tex]

Explain how the quotient of powers was used to simplify this expression

Answers

Answer:

See explanation

Step-by-step explanation:

Given:

5⁴/25 = 5²

25 = 5²

Then,

5⁴ / 25

= 5⁴/5²

Note:

• multiplication sign means addition in indices

• Division sign means subtraction in indices

Both numerator and denominator have the same base, so you'll pick one of the bases and subtract the powers

So,

5⁴/5²

= 5^(4 - 2)

= 5^(2)

= 5²

Therefore,

5⁴ / 25 = 5²

On the unit circle, which of the following angles has the terminal point
coordinates.
A. 45
B. 135
C. 225
D. 315

Answers

Answer: C. 225

Step-by-step explanation:

help and explain//////////////////////////////////////////////

Answers

Answer:

(f + g)(x) = 5x - 3

Step-by-step explanation:

(f + g)(x)

= f(x) + g(x)

= 2x + 4 + 3x - 7 ← collect like terms

= 5x - 3

Answer:

5x - 3

Step-by-step explanation:

(f + g)(x) = f(x) + g(x)

= 2x + 4 + ( 3x - 7 )

= 5x - 3

(x+4y)^(6)
Can Someone please me how to do this PLEASE!

Answers

The answer is x+4y=-6

If a bus travel for 120 minutes at a speed of 75 kilometers per hour how far has the bus traveled?

Answers

Answer:

150 km

Step-by-step explanation:

Put the minutes into hours 120min is 2 hours.

Distance = speed * time

Distance = 75 * 2

Distance = 150

Answer:

150 kilometers

Step-by-step explanation:

if the bus is going 75 kilometers an hour and they traveled for 120 minutes (exactly 2 hours) then you would just multiply 75 by 2 to get 150 kilometers total.

What is the value of x?

Answers

Answer:

D. 30°

Step-by-step explanation:

By exterior angle theorem:

x° + 70° = 100°

x° = 100° - 70°

x° = 30°

x = 30

solve for x. Round to the nearest tenth, if necessary.​

Answers

Answer:

7.1

Step-by-step explanation:

We used SOHCAHTOA because it's a right angle triangle

So because we have an angle with an adjacent of 6.3 and hypotenuse of x

We will use

Cos=adjacent /hypotenuse

7.1 like the last answer if not you can round that

Heya Kitties!
What is the value of x?

18x−16=−12x−4



x =

Answers

Answer:

2/5

Step-by-step explanation:

18x+12x=-4+16

30x=12

x=2/5

Answer:

18x-16=-12x-4

18x+12x=-4+16

30x=12

x=12/30

x=2/5

According to the number line, what is the distance between points A and B?

0 6 units
7 units
O 12 units
O 14 units

Answers

Answer:

14 units

Step-by-step explanation:

A = - 2, B = 12

Therefore,

d(A, B) = 12 - (-2) = 12 + 2 = 14 units

pls help me don't know what to do

Answers

Answer:

x=15

Step-by-step explanation:

The 60 degree angle and the (x+45) degree angle are both the same degree because they are vertical angles.

So to solve, just subtract 45 from 60

60-45=15

That's your answer!

Hope this helps!

What type of health screening would this patient most likely receive?
Sue is a 45-year-old woman with a family history of breast cancer. Her healthcare professional will most likely recommend that she receive a .

Answers

Answer:

A mammogram is what she would receive

Step-by-step explanation:

Other Questions
Which symptoms possibly indicate a major depressive disorder?Select all that apply.irritability and angry outburstsloss of interest in favorite activitiesexcessive sleeping and fatigue occasional bursts of intense energy explain with example the need of standard measurement system help me please I will mark you as brainliest and it is 29 point who cannot do this homework do not answer and you will not get the point ok grade 7 Which table represents a direct variation function? Solve for x. Round to the nearest tenth, if necessary. What is the radius of the circle: x^2+y^2=41. (0, -4)2. (0, 4) 3. (2, -4)4. (0, 0) Pls help this is rlly important!! Youll get branliest bc this is hard and Im stuck. Assume General Electric (GE) has about 10.3 billion shares outstanding and the stock price is $37.10. Calculate the market value for GE. (Approximately) 1992 governance was introduce as Lupo Corporation uses a job-order costing system with a single plantwide predetermined overhead rate based on machine-hours. The company based its predetermined overhead rate for the current year on the following data: Total machine-hours Total fixed manufacturing overhead cost Variable manufacturing overhead per machine-hour 31,400 $219,800 4 Recently. Job T687 was completed with the following characteristics: Number of units in the job 10 Total machine-hours Direct materials Direct labor cost $ 580 $1,160 The total job cost for Job T687 is closest to: (Round your intermediate calculation to 2 decimal places) Graph a line with a slope of 4 that contains the point (3,0). FOR 100 POINTS PLSI DONT NEED THE EQUATION I NEED TO SEE IT GRAPHED ALSO PLEASE PLEASE HELP ME What is the first step when applying properties of operations to divide 73.85 by 4.1? Sean is a bookworm. On average, he reads pages in minutes. a. Do students and adults have the same rights?b. Does this seem fair to you?c. Why or why not?d. Give ONE reason that it might be important for students to have limited rights Cardinality ratios often dictate the detailed design of a database. The cardinality ratio depends on the real-world meaning of the entity types involved and is defined by the specific application. For the binary relationships below, suggest cardinality ratios based on the common-sense meaning of the entity types. Clearly state any assumptions you make. Entity 1 Cardinality Ratio Entity 21. Student SocialSecurityCard2. Student Teacher3. ClassRoom Wall4. Country CurrentPresident5. Course TextBook6. Item (that can be found in an order) Order7. Student Class8. Class Instructor9. Instructor Office10. E-bay Auction item E-bay bid Ngun lc ca nn kinh t bao gm HELP PLSSSS I will GIVE BRAINLYEST!!!!!!!!!!!!!!!!!!!!!!!!!!!!!!!!!!!!!!!!!!!!!!!!!!!!!!!!!!!!!!!!!! Use a spelling word to complete the sentence.Paul was eager, or , in preaching of the Gospel. After heating a fixed volume of gas at 50 psia from 300 R to 600R, its pressure will be: Ed decided to build a storage box. At first, he was planning to build a cubical box with edges of length n inches. To increase the amount of storage, he decided to make the box 1 inch taller and 2 inches longer while keeping its depth at n inches. The volume of the box Ed built has a volume how many cubic inches greater than the box he originally planned to build?